LSAT and Law School Admissions Forum

Get expert LSAT preparation and law school admissions advice from PowerScore Test Preparation.

 ChicaRosa
  • Posts: 111
  • Joined: Aug 23, 2016
|
#28173
My understanding of this question is that it's a "What role does this statement play in the argument" question type and I want to make sure I'm understanding the structure of the argument?

Premise: Police reports show that at least 70% of people who commit crimes are caught.

Sub-conclusion: All burglars are criminals, so although some burglars will undoubtedly escape, a large percentage of them will be caught.

Conclusion: The statement in last Thursday's editorial about someone who commits a burglary runs almost no risk of being caught is clearly false.

I was stuck between D and E and I ended up choosing D instead of E (the correct answer). And I want to know why E is correct and not D?

I thought E was wrong because I thought it was supposed to undermine the conclusion so I ended up choosing D instead but I don't understand what the wording of D means is it saying that it's the main conclusion?

Here's my reasoning why I eliminated the other answer choices:

A) Wrong! It supposedly undermines the sub-conclusion and the editor is trying to counter-attack the claim.

B) Wrong! It's a premise that is part of a sub-conclusion!

C) Is Wrong! There is no example of a case under discussion.


Can anyone help me?

Thank you!
 Shannon Parker
PowerScore Staff
  • PowerScore Staff
  • Posts: 147
  • Joined: Jun 08, 2016
|
#28447
Hi there,

You are correct that the question is asking you to identify what role the statement plays in the author's argument.

Answer choice D is saying that the claim "that some burglars will undoubtedly escape" is the position that the person who wrote the letter to the editor is saying is false. However, the position that the writer is saying is false, is not that some burglars will get away, it's that burglars run "almost no risk of being caught," i.e.. that they will almost ALL get away.

The writer is saying the exact opposite, that although some will get away, most (a large percentage) will be caught. The writer is simply conceding that some will get away, without undermining the conclusion that most will get caught. Which is answer choice E.

Hope this helps.

~Shannon

Get the most out of your LSAT Prep Plus subscription.

Analyze and track your performance with our Testing and Analytics Package.